You are on page 1of 55

1

Neuroradiology Boards Questions


The following are True/False questions:

1995
Which of the following masses are hyperdense on non-contrast-enhanced CT?
1. medulloblastoma
2. acoustic neuroma
3. primary lymphoma
4. prolactinoma
*
Answer: 1. true 2. false 3. true 4. false

Danhert lists medulloblastoma, lymphoma, and pituitary adenoma as hyperdense lesions.


-- probably secondary to calcification, hemorrhage, or dense protein. p 139
Medulloblastoma on NCCT. are hyperdense, well circumscribed lesions. They have
occational calcification and cytic degeneration. p 85
The classic teaching is that lymphoma is typically hyperdense on NCCT at enhances to a
moderate degree. This can be a little more variable in the HIV population. p 92
On CT , a microadenoma is hypodense compared to normal gland. Macroadenomas have
the same signal characteristics but have a higher propensity for hemmorrhage and infarction
because of the marginal blood supply. p314 requisites, 442 Lee
Acoustic neuromas can be seen on CT with contrast because it diffusely enhances.
Intracanilicular acoustics are difficult to ID on CECT. Basically, CT is best for ID bone erosion.
p 493
ref: Danhert, p 139, Requisites, p 313-4, 85, 92, p 442 , 493Lee and Rao

1993 c PreTest
The pterygopalatine fossa communicates with the:
5. infratemporal fossa via the pterygomaxillary fissure
6. oral cavity via the pterygopalatine canal
7. orbit via the superior orbital fissure
8. middle cranial fossa via the foramen ovale
9. inferior meatus of the nasal cavity via the sphenopalatine foramen
*
Answer: 5. True 6. True 7. False 8. False 9. False

The pterygopalatine fossa is a small pyramidal space situated below the orbital apex. It is
extremely important, as it communicates with multiple fissures, canals, and foramina, including:
1. the infratemporal fossa, via the pterygomaxillary fissure
2. the oral cavity, via the pterygopalatine canal
3. the orbit, via the inferior orbital fissure
4. the middle cranial fossa, via the foramen rotundum and the vidian canal
2

5. the superior meatus of the nasal cavity, via the sphenopalatine foramen

The pterygopalatine fossa contains the pterygopalatine ganglion and the distal portion of
the internal maxillary artery.
It is obvious that once a malignant neoplasm reaches the pterygopalatine fossa, it may
easily spread to a wide variety of anatomic locations.
Reference: Harnsbarger, pp. 390-391, Requisites, p 383
*

1995
Regarding parotid gland neoplasms:
10. Warthin’s tumor commonly has perineural spread
11. Warthin’s tumor is the most common parotid gland neoplasm
12. pleomorphic adenomas are typically medial to the facial nerve
13. pleomorphic adenoma is more common in women
another question on spread???
*
Answer: 10. False 11. false 12. false 13. true

80% of tumors are benign, 80% are pleiomorphic adenomas, 80% are in the superficial
lobe of the parotid, 80% of pleiomorphics are in the parotic, 80% of pleiomorphics if left unRx
'ed stay benign.
The portion of the parotid that extends deep to the plane of the facial nerve is the deep
lobe. The superficial lobe extends from just under the skin. These lobes don't really exist but are
arbitrary distinction for surgical purpose. If the tumor is in the superficial lobe, the facial nerve
is dissected out deep to the mass and the tumor is resected superficial to the facial nerve. Are
most commonly seen in middle aged women. p 414, 420
Warthin's tumor is nearly exclusive to the parotid and is the most common multiple and
bilateral tumors in the gland. These lesions are entirely benign. Warthins are most commonly
seen in elderly men. p 420 They are the second most common benign tumor of the parotic-- 10%
as stated in Danhertp 256. These lesions are usually round to oval encapsulated masses. p 824
Robbins
ref: Requisites p 420, 414, Danhert p 256, Robbins, p 824

1995
Regarding Neurofibromatosis Type I
14. acoustic neuroma
15. sphenoid wing dysplasia
16. optic nerve glioma
17. macrocrania
*
Answer: 14. false 15. true 16. true 17. true

NF-1 is von Recklinghausen's disease. To dx must have two or more of the following
findings-- >6 cafe au lait spots, >2 lisch nodules of the iris, >one plexiform neurofibroma,
3

axillary freckling, >one bony dysplasia, pseudarthrosis of long bone, optic glioma, 1st degree
relative with NF. Acoustic neuroma is associated with NF-2.
ref: Requisites p 266
The orbit frequently displays a characteristic unilateral defect fo the greater and lesser
wings of the sphenoid attributed to underlying mesodermal dysplasia. Other cranial abnormalities
include bone defect along left side of the lambdoid suture, hypoplastic maxillary and ethmoid
sinuses. Themandible and maxilla may be affected by overlying soft tissue abnormalities. p
1220 Resnick.
Macrocranium is seen in 75%. Manaster p 326

1995
Which of the following are migrational abnormalities?
18. schizencephaly
19. heterotopic bands
20. alobar holoprosencephaly
21. Chiari II malformation
22. unilateral megaencephaly
*
Answer: 18. true 19. true 20. false 21. false 22. true

Schizencephaly is a abnormality of neuronal migration is the 5-7th week. It is a cleft seen


coursing from the ependyma to the pial surface. The disorder is secondary to failure of the
germinal matrix to form. p 253
Heterotopia is usually in the gray matter which is located in the wrong place due to arrest
in neuronal migration. This usu occurs in the 7th to 16th week when migration of the neuroblasts
form the perivent to the pia is thwarted. Ectopic gray matter is usually in the subependymal or
subcortical location. p 256
Holoprosencephaly is a congenital abnormality where separation of the right and left
cerebral hemispheres is incomplete. Alobar type is the most severe and demonstrates almost no
separation of the cerebral hemispheres and ventricles. There is one large horseshoe shaped vent
with no interhemispheric fissure. The BG and thalamus are fused. p 252
Etiologies of unilateral cranial enlargement-- Dyke-davidoff-masson syndrom,
hemimegalencephaly(neruonal migrational anomaly), NF, Klippel-Trenaunay Synd, Proteus
Sydrome. Can get polygyria associated with hemispheric size, or agyria associated with less sever
hemispheric enlargment. MR demonstrates distorted thickened cortex with ipsilateral ventricular
dilatation. p 258
Chiari malformation is dx when cerebellar tonsils are below the foramen magnum. ? no
etiology stated. p 261

ref: Requisites
4

1995, 1992, 1989


Infarct of the posterior cerebral artery affects:
23. ipsilateral thalamus (95)
24. contralateral thalamus (95)
25. most anterior superior cerebellum (95)
26. posterior aspects of temporal and parietal lobes (95)
27. frontal horn regions
28. head of the caudate (95)
29. contralateral cerebral peduncle (95)
*
Answers: 23. True 24. False? 25. False 26. True 27. False 28. false 29.
???false- can affect peduncle but contralateral??

The important thing to know is that the posterior cerebral artery supplies the thalamus via
thalamic perforators. However, there is also cross flow to contralateral as well as ipsilateral
sides. Therefore, with PCA occlusion you can infarct no thalamus, ipsilateral thalamus,
contralateral thalamus, or ipsilateral and contralateral thalamus. Most commonly, the ipsilateral
thalamus is infarcted and only rarely the contralateral thalamus. The PCA does not supply the
cerebellum. It supplies occipital as well as posterior temporal and parietal lobes as well as
choroid plexus and ependyma of the third and lateral ventricles. The frontal lobe is the domain
of the ACA.
Midbrain perforating arteries arise from the basilar and prox PCA. Infarctions of these
arteries can affect CN III, IV, causing oculomoter deficits; the cerebral peduncles- affecting
motor strength, medial lemniscus- altering sensation; the red nucleus and substantia nigra- affects
coordination and motor control,; reticular activation substance- affect level of consciousness.
The PCA have small premamillary thalamoperforate, and thalamogeniculate branches
supplying the hypothalmus, midbrain, inferior thalamus. Infarctions of these vessels may affect
memory and emotion- anterior thalamus, endocrine- hypothal, language- pulvinar, pain
sensation- thalami, sight- lateral geniculate, motor control- subthalamic nuclei.
The next branches of the PCA are the medial and lateral posterior choroidal arteries that
supply the trigone of the lateral vents. They supply the tectum, choroid plexus of the 3rd vent,
and the thalami. The lateral posterior choroidal arteries supply the choroid of the lateral and third
vents and the posterior thalamus , the fornix.( fornix lies anterior and inferior to the corpus
collosum)
The PCA continues branching off the anterior and posterior temporal ateries and posterior
pericollosal artey before terminating into the parietooccipital branches and calcarine arteries.
PCA infarct affects vision- calcarine, memory- post-inferior temporal lobe, smell- hippocampus,
emotion-splenium.
One named branch off the ACA is the recurrent artery of Heubner which supplies the
head of the caudate and anteroinferior internal capsule. p 57

ref: Requisites p 61-2


5

1995
Regarding idiopathic orbital pseudotumor:
30. insiduous onset
31. primary therapy is radiation therapy
32. often involves the lacrimal gland
33. can extend intracranially
34. the cause is often unknown
*
Answer: 30. false 31. false 32. true 33. false -- it would be Tolosa-hunt 34. prob
true-- may be autoimmune

= IDIOPATHIC INFLAMMATORY PSEUDOTUMOR


= nongranulomatous inflammatory process affecting all intraorbital soft tissues
Etiology:
(a) cause not apparent at time of study: bacterial, viral, foreign body
(b) systemic disease presently not apparent: sarcoidosis, collagen, endocrine
(c) idiopathic: probably abnormal immune response
Incidence: 25% of all cases of unilateral exophthalmos; most common cause of an intraorbital
mass lesion in adult
Age: young female
Histo: lymphocytic infiltrate
May be associated with: Wegener granulomatosis, sarcoidosis, fibrosing mediastinitis,
retroperitoneal fibrosis, thyroiditis, cholangitis, vasculitis, lymphoma
• unilateral painful ophthalmoplegia
• proptosis, chemosis, lid injection
• limitation of ocular movement
Location: retrobulbar fat (76%), extraocular muscle (57%), optic nerve (38%), uveal-scleral area
(33%), lacrimal gland (5%)
(a) Tumefactive type (common)
Ö discrete / poorly defined intra- / extraconal mass
= "pseudotumor" close to surface margin of globe
(b) Myositic type (unusual)
Ö enlargement of one / more extraocular muscles close to insertion in globe with ill-defined
margins Ö typically involves muscles + tendon insertions (DDx to Graves disease with muscle
involvement only)
Ö increased density of retro-orbital fat (may involve anterior compartment)
Ö thickening and enhancement of sclera near Tenon capsule
Ö enlarged lacrimal gland
Ö proptosis
MR:
Ö lesion isointense to fat on T2WI
Prognosis:
(1) remitting / chronic + progressive course
6

(2) rapid dramatic + lasting response to steroid therapy


DDx:
lymphoma (may be confused with lymphoma clinically, radiographically, pathologically),
thyroid ophthalmopathy (tapering of distal muscles, painless proptosis), radiation therapy

Orbital pseudotumor is a common cause of unilateral exophtalomos. Clinical features


include restriction of ocular motility, chemosis, lid swelling, pain. They usually have rapid onset
and repond to steroids. There is a chronic progressive from which responds poorly to steroids.
and is treated with chemo and XRT. In the acute case, it is a inflammatory condition that may be
an autoimmune condition involving the lacrimal gland, extraocular m, connective tissue
surrounding the dura of the optic nerve, the orbital fat, epibulbar connective tissue, and sclera.
May present as a lacrimal mass.
Tolosa-Hunt syndrome is a inflammatory condition similar to orbital pweudotumor that
involves the cavernous sinus and orbital apex. This presents with painful opthalmoplegia. p
297
ref: Requisites

1995
Which of the following would be seen by MR in a patient who is 1 day S/P CVA?
35. hemosiderin deposition
36. mass effect
37. meningeal enhancement
38. parenchymal enhancement
39. vascular enhancement
*
Answer: 35. false 36. true 37. false 38. false 39 true

Findings on MR may be seen within a few hours. Swelling is observed on T1 and proton
density w/o changes of signal. 8hrs, high signal develops on T1 and PDWI. 16hr, low signal on
T1. Arterial enhancement which is slow flow is detected in approximately 50% of ischemic
lesions. Arterial enhancement is more sensitive than T2 in the first few hr of stroke.
Reappearance of flow void occurs around 7-11days. this suggests establishment of collateral
flow of lysis of embolus.
Noncortical infarctions have parenchymal enhancement between 4-7 days whereas
cortical infarctions enhance after 6 days. Enhancement of the infarct may last 6-8weeks.
On CT, 6hr-faint loss of gray-white border, 12-24hr, indistinct low density, feeble mass
effect, >24hr, more mass effect, 3-5days, peak mass effect. Parenchymal enhancement is rarely
seen before 3 days. If see earlier enhancement, either because of excellent collaterals, or embolus
has moved distally. Mass effect resolves at around 2-4 weeks.
ref: Requisites p 112

1994
Which of the following are causes of third cranial nerve palsy?
40. diabetes
7

41. multiple sclerosis


42. posterior inferior cerebellar artery aneurysm
43. migraine
44. lymphomatous meningitis
*
Answer: 40. True 41. True? Not typically 42. True 43. false 44. True

a, b, and e seem to be certain. Jill thought that migraine might be correct.


Diabetes classically gives a pupillary-sparing third-nerve palsy because it does not
involve the parasympathetic fibers which control pupillary constriction.
An unruptured posterior communicating artery (not the posterior inferior cerebellar)
anuerysm classically gives an isolated and complete third-nerve palsy with a dilated pupil.
Meningitis is the most common of the above listed.
Reference: Atlas, pp. 396-408
*

1994
Which of the following are potential causes of enhancing posterior third ventricular mass?
45. dysgerminoma
46. vein of Galen aneurysm
47. pineal cyst
48. PNET
49. pinealcytoma
50. chemodectoma
51. colloid cyst
*
Answer: 45. True 46. True 47. false 48. True ??? (see above) 49. True 50. ??? 51.
False (shouldn’t enhance)

Pineal cysts typically ring enhance on MRI. Supratentorial PNETs are distinctly
uncommon and in her tenure at CHOP Jill has not seen one involving the pineal. We are
therefore not including this. If you include pineoblastomas as a PNET( table 3-6 p 83), the
answer is easy with PNET being true as well.
Pineal region masses include germinoma aka dysgerminona or seminoma which have
marked enhancement and are almost exclusively male, pinealblastoma and pineocytoma.
Pineocytoma enhance vividly.
Other lesions in this area include pineal and tectal gliomas, cavernous hemangioma,
meningioma, and benign cyst. Pineal cyst can be large. Pineocytoma can be hard to differentiate
froma cyst because it can also have cystic components. Key is long-term f/u. Because pineal
cyst are surrounded by limbs of inernal cerebral veins, have to be careful not to misread vascular
enhancement as solid mass enhancement.
ref: Requisites p 96-7

1994
8

Regarding pituitary microadenoma, which of the following are true?


52. at presentation most are <2 cm in diameter
53. prolactin-secreting adenomas are the most common hormonally active adenomas
54. extension into the cavernous sinus causes extensive amounts of prolactin secretion/excretion
55. bromocryptine treated patients commonly hemorrhage in the pituitary fossa
56. usually centrally located in the pituitary
*
Answer: 52. False--all NOT most 53. True 54. False 55. False 56. false

From Osborne, page 467


By definition all microadenomas are less than 1 cm
There are many microadenomas discovered as incidental findings at autopsy therefore
most are not prolactin-secreting.
Pituitary adenomas are the most common intrasellar tumor, representing 10-15% of all
intracranial neoplasms. Prolactinomas account for 40-50% of hormonally active adenomas; less
commonly, adenomas may produce growth hormone (20%) or ACTH (20%). T1-weighted
images are best for visualizing the pituitary because the bright signal of the surrounding CSF on
T2-weighted images obscures the gland. Hemorrhage into macroadenomas is not uncommon.
Reference: Atlas, pp. 632-640
Treatment with bromocriptine increases the likelihood of hemorrhage which has been
reported in more than half of the bromocriptine treated adenomas. ? Hemmorrhage into the
gland not into the fossa. p 315 Requisites
*

1993, 1988
Differential diagnosis of a cerebellopontine angle mass in a child includes:
57. epidermoid
58. facial nerve neuroma
59. cerebellar astrocytoma
60. PNET
*
Answer: 57. True 58. True 59. False 60. False

Most pediatric cerebellar astrocytomas arise in the midline


A PNET is usually seen in the midline arising from the superior vellum of the 4th
ventricle in kids. In young adults they are more eccentric.
The Differential Diagnosis (in kids):
1) Acoustic Schwannoma - 75%
2) Meningioma - 10%
3) Facial Nerve Schwannoma -4%
4) Aneurysm (vertebral, basilar, PICA)
5) Brain stem glioma - most common presentation is multiple cranial nerve palsies. Most
commonly occurs in the pons.
6) Arachnoid cyst
9

7) Epidermoid ??? - congenital lesions of ectodermal origin. They are characteristically lobular
and hypodense on CT and do not enhance.

others: paraganglioma, metastases, subarachnoid tumor spread, lipoma, hemangioma, choroid


plexus papilloma, ependymoma
*

1993
A hemosiderin ring is:
61. best seen on CT
62. dense on non-contrast enhanced CT
63. dark on T2 weighted MR
64. bright on T1
65. brighter after the administration of gadolinium (T1 weighted images)
*
Answers. 61. False 62. False 63. True 64. False 65. False

Hemosiderin rings are low density on CT.


Hemosiderin rings are dark on T1 and T2.
PUT IN EVOLUTION OF HEMORRHAGE ON MR AND CT!!!
*

1993
Which of the following cause a cystic midline mass that can mimic an enlarged third or fourth
ventricle?
66. dermoid cyst
67. intraventricular cysticercosis
68. hypothalamic polycystic astrocytoma
69. suprasellar arachnoid cyst
70. cerebellar astrocytoma
*
A dermoid cyst is a midline mass but is rare in the suprasellar cistern. Its capsule is
thick with peripheral calcification ( p 320 Requisites). The presence of fat or calcium in it can
distinguish it from a large ventricle.
Cysticercosis is one of the most common parasitic diseases that affect the brain. “A
large cyst or cluster of cysts within the 4th ventricle may simulate dilatation of that ventricle,”
Lee, Rao, and Zimmerman 1992, pp. 572-573. Cysticercosis is acquired by ingestion of a
parasite which is in insufficiently cooked pork. The cysticerci are located in brain
parenchyma, subarachnoid space, and the ventricles (rarely intraspinal). The parenchymal
cysts have a propensity for cortical and deep gray matter; whereas subarachnoid cysts can
produce hydrocephalus and mass lesions, particularly in the suprasellar cistern,
cerebellopontine angle cistern, and the Sylvian cistern. There are four stages of the cyst. In
stage I, the larvae are alive and the cyst has clear fluid. Edema is minimal and the cyst is
surrounded by a thin capsule. On MR, the fluid of the cyst is isointense on all pulse sequences
and the eccentric scolex (which appears as a mural nodule) can be identified. CT shows a
circumscribed cyst with density similar to CSF (the scolex is more dense). Visualization of the
10

scolex is pathognomonic of cysticercosis. When the larvae die, the cysts incite an
inflammatory response.
The hypothalamic polycystic astrocytoma has incomplete necrosis which sometimes
leads to a polycystic appearance. Supratentorial astrocytomas in children frequently involve
the thalamus.
If a suprasellar arachnoid cyst invaginates into the 3rd ventricle, it can be mistaken for
an enlarged 3rd ventricle on the basis of acqueductal stenosis.
Most cerebellar astrocytomas arise in the midline and can be cystic. Cystic
astrocytomas with a mural nodule account for 50% of cerebellar astrocytomas in children.
*
Which of the following cause a cystic midline mass that can mimic an enlarged third or fourth
ventricle?
66. dermoid cyst
67. intraventricular cysticercosis
68. hypothalamic polycystic astrocytoma
69. suprasellar arachnoid cyst
70. cerebellar astrocytoma

Answers: 66. False 67. True 68. True 69. True 70. True

1993
Encephaloceles are associated with which of the following?
71. skull defect
72. syndactyly
73. echogenic kidneys
74. pyelectasis
75. microcephaly
*
An encephalocele arises from failure of dorsal induction, which normally occurs at 3-4
weeks gestational age. However, basal (sphenoidal) encephaloceles arise as a result of abnormal
ventral induction (2 months gestational age). An encephalocele is extracranial extension of
leptomeninges and brain tissue (meningocele is extracranial extension of leptomeninges alone).
Encephaloceles are much more common than meningoceles and typically occur along the
midline.
Nervous system anomalies which are frequently associated with encephaloceles: Dandy-
Walker malformation and Chiari III malformation (occipital), dysgenesis of the corpus callosum
(parietal), dysgenesis of the corpus callsum, callosal lipomas and globe anomalies such as
microphthalmia/anophthalmia (frontoethmoidal), and agenesis of the corpus callsum and optic
nerve dysplasias (basal).
A skull defect can give rise to a meningoencephalocele - 70% are occipital, 10% parietal,
and 9% frontal (in the U.S.). Southeast Asian women have a propensity for nasofrontal or
sphenoethmoidal meningoencephaloceles.
Meckel-Gruber syndrome consists of renal cysts (micro would be echogenic),
microcephaly, polydactyly, and meningoencephaloceles.
11

Amniotic band syndrome also gives multiple irregular asymmetric off-midline


encephaloceles (among abnormalities found in amniotic band syndrome is distal syndactyly).
I couldn’t find any association with pyelectasis.
References: Lee, Rao, and Zimmerman 1992, pp. 194, 202-204; Dahnert 1993, p. 169, 629
*
Encephaloceles are associated with which of the following?
71. skull defect
72. syndactyly
73. echogenic kidneys
74. pyelectasis
75. microcephaly

Answers: 71. True 72. True 73. True 74. False 75. True

1993
Which of the following are true with regards to amyloid angiopathy?
76. is associated with hemorrhage of the basal ganglia
77. associated with hemorrhage of the brainstem
78. associated with hemorrhage in the cerebral hemisphere
79. is seen most commonly in people with systemic hypertension
80. most commonly seen in the elderly
*
Amyloid angiopathy = Deposition of amyloid in the media and adventitia of small and
medium-sized vessels of the superficial layers of the cerebral cortex and leptomeninges with
sparing of the deep gray nuclei. Amyloid accumulates Congo red dye and exhibits yellow-green
birefringence when examined with polarized light. Hemorrhages are usually lobar, involving
frontal and parietal lobes including subjacent white matter. Subarachnoid and subdural
hemorrhages have also been reported. There is a propensity for recurrent hemorrhages or
multiple simultaneous hemorrhages. The hemorrhages are rarely found in the cerebellum, white
matter, basal ganglia, and brain stem. The major risk factor is aging, but doesn’t correlate with
hypertension. It is not associated with systemic amyloidosis.
Reference: Grossman & Yousem 1994, pp. 132-133
*
Answers: 76. False 77. False 78. True 79. False 80. True

1994
Symptoms associated with L4 radiculopathy include:
81. hip pain radiating to the lateral thigh
82. gluteal pain
83. absent knee jerk reflex
84. absent ankle jerk reflex
85. hamstring weakness
*
12

L4 radiculopathy should present as hip pain radiating to the anterior thigh (not lateral),
knee, and medial calf. Sensation is impaired over the medial calf and the territory of the
saphenous nerve. One gets an absent knee jerk although overlapping innervation may making it
difficult to detect. Muscle weakness is difficult to establish owing to the heavy overlap of root
innervation. The quadriceps, abductor longus, gluteus medius, and tibialis anterior muscles may
be weak.
*
Answer: 81. False?? 82. False 83. True 84. False 85. False

1994
Regarding gliomas:
86. 70-80% of gliomas in children originate in the brain stem
87. 80% of low grade lesions eventually degenerate into high grade lesions
88. there is a poor prognosis associated with pilocytic astrocytoma with 10-20% ten year survival
89. pilocytic neoplasms are typically lobular, well-marginated, and not calcified
90. the most common presenting age for gliomas is approximately 50 years old
*
13

Less than 70-80% of childhood gliomas are from the brain stem. Junvenile pilocytic
astrocytomas are the most common infratentorial neoplasm in the juvenile age group. p 83
Requisites.
Less than 80% of low grade gliomas will degenerate if you include things like pilocytic
astrocytoma.
C is obviously false.
Osborne states on page 407 that the peak incidence of gliomas is in the seventh decade.
*
Answer: 86. False 87. False 88. False 89. True 90. False

1994, 1992, 1989v


On an injection of contrast into the left vertebral artery, there is non-visualization of the
ipsilateral posterior cerebral artery. Which of the following are possible causes?
91. origin from the left internal carotid artery
92. occlusion of the left posterior cerebral artery
93. spasm
94. flow phenomenon
95. aplasia
*
Carotid-basilar anastomoses are persistent embyological anastomoses.
Possible carotid ---> basilar anastomoses:
1) persistent trigeminal artery (most frequent, discovered in 0.1-0.2% of cerebral angiograms) -
arises just before the cavernous segment of the ICA.
2) otic artery
3) hypoglossal artery
4) suboccipital artery
5) proatlantal intersegmental artery
6) posterior communicating artery
*
Answer: 91 True -fetal origin of PCOM 92. True 93. True 94. ??? 95. ???

1993
Which of the following are true regarding CSF rhinorrhea?
96. mucus retention cyst of the frontal sinus can be a cause
97. osteoma of the frontal sinus can be a cause
98. medial blow-out fracture of the orbit is a cause
99. most commonly involves the sphenoid sinus
100. temporal lobe arachnoid cyst can be a cause
*
14

An osteoma is a bony mass which occasionally results in mucocele and/or


pneumocephalus as the posterior wall of the frontal sinus is breached.
A medial wall blow-out fracture involves the lamina papyracea of the ethmoid bone
Traumatic CSF rhinorrhea most commonly is secondary to fractures through the
frontoethmoidal complex and middle cranial fossa. The most common cause of CSF rhinorrhea
is trauma.
Reference: Dahnert 1993, p. 124, 127
Can arise from postoperatively when the cribiform plate is violated. Usually get post
ethmoidectomy/ p 368 Requisites.
Pneumocephalous is found in 20% of pt with CSF leaks. Blood clot forms around the fx
initially, but when the clot resolves, CSF leak begins. Usually the CSF leaks stops within the 1st
week, 99% w/in 6wks. Post trauma CSF leaks most often involve the frontal, ethmoid sinus, and
cribiform plate. Less frequently, the sphenoid sinus, petrous and other calvarial sites are
involved. p516 Lee, Rao, Zimm.
Internal orbital wall blowout involves fx of ethmoid or maxilllary sinus Medial wall fx ar
characterized by the passage of air thru the fx defect into the orbit giving orbital emphysema.
The air gains acces w/ increased intranasal pressure such as nose blowing. Medial blow out fx
characteristically involves the medial floor and the medial orbital wall, and medial wall of the
maxillary antrum. Clinical signs of blow out include enopthalomos, diploplia, limited upward
and outward gaze.p 51 Harris and Harris
*
Answers: 96. False 97. True 98. ???false 99. false 100. False

1995 ITE
Concerning the use of contrast material:
101. approximately 18 cc is the dose for an internal carotid artery angiogram
102. approximately 30 cc of standard nonionic contrast is the dose for lumbar myelography
103. approximately 4 cc is the dose for an external carotid artery angiogram
104. approximately 25 cc of standard nonionic contrast is the dose for cervical myelography
105. nonionic and ionic contrast material exhibit a similar degree of nephrotoxicity
*
Internal carotic artery injection is about 6ml/s for a total of 8cc. External carotid artery
injection is 3ml/s for 5cc. p 16
In general, 3.06g limit for (non)iodinated contrast is instilled intrathecally. Therefore,
10cc of 300mg/ml concentration of 12cc of 240mg/ml concentration is injected for lumbar or
cervical myelography.--from package insert of Omnipaque. If doing a cervical myelography via
a lumbar approach, the max dose should be given. p 18
Iodinated contrast agents are potentially nephrotoxic with decreases in creatinine
clearance. The effect on creatinine clearance seems to be less with nonionics. The potential for
renal damage is greatest in pt with predisposing renal disease/illness.
Ref: Requisites. p16-18

Answer: 101. false 102. false 103. prob true 104. false 105. ???
15

1993
Which of the following are frequently calcified lesions in the posterior fossa?
106. dermoid
107. astrocytoma
108. medulloblastoma
109. hemangioblastoma
110. acoustic neuroma
*
Dermoids are midline lesions containing fat, squamous epithelium, hair follicles, sweat
glands, and sebaceous glands. The capsules are thick with peripheral calcification. They rarely
arise in a suprasellar location. They are more frequent in the posterior fossa than
supratentorially.
[Astrocytoma] A GBM is common in the posterior fossa of an adult. They are associated
with a large amount of edema and mass effect. Central necrosis is a hallmark. Posterior fossa
astros uncommonly calcify--<10%. p 84 Requisites
Medulloblastomas have a 10-21% incidence of calcifications in adults and in kids is 10-
15% and listed as uncommon p 84, 82 Req. 10-20% are cystic in children (adults, it is cystic 60-
80%).
A hemangioblastoma is stereotypically a cystic mass with a solid mural nodule (55%)
which is highly vascular and has serpentine signal voids of feeding vessels. 40% are solid.
Acoustic neuroma (Schwannoma is a more accurate term) occurs most commonly along
the 8th cranial nerve. On MR they are usually isointense to slightly hypointense compared with
pontine tissue (all pulse sequences). They enhance. Edema is present in one third. Hemorrhagic
and cystic changes are unusual.
ref: Requisites
*
Answers: 106. True 107. False 108. True- false - uncommonly calcifies 109. False
110. False

1993
Which of the following is associated with the shaken baby syndrome?
111. retinal hemorrhage
112. subdural hematoma
113. cortical hematoma
114. white matter shear injury
*
In shaken baby syndrome retinal hemorrhages are observed with subdural (particularly
interhemispheric) hematomas and/or subarachnoid hemorrhage. Contusions and diffuse cerebral
swelling are also noted. The abused child can have epidural, subdural, subarachnoid, and
intraventricular hemorrhages as well as DAI.
Traumatic subdural hematomas in the child (from any traumatic cause) are bilateral in
80%.
16

*Answers: 111. True 112. True 113. True 114. True

1993
Which of the following are associated with increased incidence of primary brain tumor?
115. von Hipple-Lindau
116. Turcot’s
117. Gardner’s
118. tuberous sclerosis
119. sickle cell disease
*
Von Hipple-Lindau syndrome is autosomal dominant and consists of cerebellar
hemangioblastomas and retinal angiomatosis (67%), pheochromocytoma, cysts of kidneys,
pancreas, liver, and epididymis, and a high incidence of renal cell carcinoma.
Turcot’s is associated with CNS tumors, especially supratentorial glioblastomas.
Gardner’s consists primarily of colonic polyps, osteomas, and soft tissue tumors.
The intracranial manifestations of tuberus sclerosis include periventricular subependymal
nodules, cortical and subcortical peripheral tubers, white matter lesions, and subependymal giant
cell astrocytomas.
In sickle cell disease the incidence of cerebrovascular occlusive disease is 5-17%.
Infarction is most likely related to stasis and ischemia in the vasa vasorum which leads to intimal
and medial hyperplasia and subsequent narrowing and occlusion of internal carotid or proximal
anterior and middle cerebral arteries.
*
Answers: 115. True 116. True 117. False 118. True 119. False

“90% of passing the boards is mental, and the other half is physical,” paraphrased from Yogi
Berra.

1992
Examples of spinal dysraphism include:
120. diastomatomyelia
121. myelolipoma
122. sacralization of L5
123. syringobulbia
124. Dandy-Walker cyst
*
For b. lipoma is associated with a tethered conus which is associated with mild
dysraphism.
The mechanism for development of spinal dyraphism is lack of folding of primodial
neural tissue into a tube. No bone is induced directly over the neural tissue. Get meningocele,
myelocele, myelomeningocele are tissue that go through a boney spinal defect. when fat,
meninges, cord roots protrude through the defect its called a lipomyelomeningocele.
17

Lipomyelomening is associated with tethered cord and Chiari II and syringohydromyelia. p


273Req
Dandy Walker is partial or complete absence of the vermis with dilatation of the fourth
ventricle. p 259
myelolipoma is a benign tumor of the adrenals. p 273 Dunnick
ref: Requisites p 259, 273, Dunnick p 273
*
Answers: 120. True 121. False 122. False 123. False 124. False

1992
Typical features of hemangioblastomas in adults include:
125. calcification
126. cystic
127. presents with hemorrhage
128. avascular
129. presents with Parinaud syndrome
*
Hemangioblastoma is the most common primary neoplasm of the adult posterior fossa
and arise most frequently in the cerebellum. It is associated with von Hippel-Lindau disease. It
does not typically present with hemorrhage although there may occasionally be hemorrhage in
the nodular component. They are very vascular with very intense staining on angiography, and
prominent flow voids on MR. It is solid in one third but commonly appears cystic +/- mural
nodule. It almost never calcifies. A spinal hemangioblastoma can present with subarachnoid
hemmorrhage. Common sx include HA, ataxia, N/V, vertigo. 40% can have P vera from
erythropoietin produation. p 81 Requisites
Perinaud’s syndrome is loss of upward gaze which is seen with pineal tumors.
Reference: Dahnert 1993, pp. 180-181
*
Answers: 125. False 126. True 127. True?? is it really a typical feature? 128. False
129. False

1992
Structures within the superior orbital fissure include:
130. oculomotor nerve
131. ophthalmic artery
132. superior ophthalmic vein
133. maxillary portion of the trigeminal nerve
134. optic nerve
*
18

The superior orbital fissure contains cranial nerves III (oculomotor), IV (trochlear), V1
(ophthalmic branch of trigeminal), and VI (abducens) as well as the superior ophthalmic vein,
sympathetic fibers, and the orbital branch of the middle meningeal artery.
The ophthalmic artery goes through the optic canal on the undersurface of the optic nerve.
The maxillary portion of the trigeminal nerve (V1) goes through the inferior orbital
fissure.
*
Answers: 130. True 131. False 132. True 133. False 134. False

1992
In a 4 y/o male a suprasellar mass, the diagnostic possibilities include:
135. dermoid
136. germinoma
137. craniopharyngioma
138. Rathke cleft cyst
139. optic chiasm glioma
140. hypothalamic hamartoma
*
The dermoid is unlikely but possible in the suprasellar area.
The germinoma is the most frequent tumor of the hypothalamic area. It is most
commonly pineal but 20% are suprasellar. The peak age is early puberty but it can be seen in a 4
y/o. M=F with suprasellar germinomas while 90% of pineal germinomas are in males. The most
common presentation of a suprasellar germinoma is the hypothalamic syndrome (diabetes
insipidus, emaciation, precocious puberty). They are relatively radio-resistant but long time
survival rates are high. There is a marked tendency for them to spread through CSF pathways -
“In the suprasellar region, the finding of an infiltrating suprasellar mass that is intrinsically
hyperdense would favor germinoma over astrocytoma.” (Seminars in US, CT, MRI. Vol.13#6,
Dec. 1992, p. 427)
A hypothalamic hamartoma usually presents before the age of 3 y/o with precocious
puberty. It is isointense to gray matter on all sequences with no enhancement. It is a rare tumor.
Craniopharyngioma is a common childhood tumor. It accounts for 50% of pediatric sellar
and suprasellar tumors (90% suprasellar). In children, they tend to be cystic and enhance. 90%
will be at least partially calcified. They extend into the anterior, middle, or posterior cranial
fossa in 25% of cases.
Optic glioma is classified as a juvenile pilocytic astrocytoma and represents 2/3 of all
primary optic nerve tumors. The mean age of presenation is 8.5 years ( 5yo in presence of NF1
and 12yo in abscence of NF). p 291 Requisites
*
Answers: 135. True 136. True 137. True 138. True 139. True 140. True
19

1992
Possible causes of pulsatile tinnitus include:
141. glomus tympanicum tumor
142. glomus jugulare tumor
143. absent carotid with carotid canal dehissence
144. cholesteatoma
145. anterior communicating artery aneurysm
146. posterior communicating artery aneurysm
*
The differential diagnosis of pulsatile tinnitus:
1. paraganglioma (including glomus tympanicum, which arises at the cochlear promontory of the
middle ear cavity, and glomus jugulare)
2. dural AVM
3. aberrant internal carotid artery
4. dehiscent or high jugular bulb
5. high-grade carotid artery stenosis
6. petrous internal carotid artery aneurysm or dissection
7. fistula of temporal bone region (cavernous carotid)
8. cholesteatoma, cholesterol granuloma
9. meningioma
10. Meniere’s disease

Cholesteatomas may be primary (congenital) or secondary (acquired). Primary


cholesteatomas arise from epithelial rests and, in the middle ear compartments, most often occur
in the epitympanum. The petrous apex is another important location. Secondary cholesteatoma
is an acquired, postinflammatory lesion which develops from ingrowth of squamous epithelium
through the eardrum. The accumulation of squamous and keratin debris within the middle ear
gradually causes erosion of the petrous bone and ossicles. Pars flaccida cholesteatomas are much
more common than pars tensa lesions; the former typically herniate into Prussak’s space in the
lateral attic. Cholesterol granulomas are postinflammatory lesions of the petrous apex that
generally occur in pneumotized petrous apices. They cause varying degrees of expansion of the
petrous apex, and their signal characteristics (hyperintensity on T1- and T2-weighted MR
images) help distinguish them from primary cholesteatomas.
Reference: Grossman & Yousem 1994, pp. 343-348
Possible causes of pulsatile tinnitus include:
141. glomus tympanicum tumor
142. glomus jugulare tumor
143. absent carotid with carotid canal dehissence
144. cholesteatoma
145. anterior communicating artery aneurysm
146. posterior communicating artery aneurysm

*
Answers: 141. True 142. True 143. True 144. True 145. False 146. False
20

1992
Sources of epidural hematoma include:
147. middle meningeal artery
148. superficial temporal artery
149. superior saggital sinus
150. diploic veins
151. vein of Galen
*
An epidural hematoma is collection of blood along the subperiosteal inner table. Itis
associated with skull fracture in 90%. It is biconvex, and since it is subperiosteal, it is limited by
the sutures (exception is when a fracture crosses a suture). Sources of blood are the meningeal
vessels (arteries (60%) or veins), dural sinuses, diploic veins, and marrow sinusoids. The
meningeal vessels are displaced from the inner table. ref: Req p 151
Collection of blood along the subperiosteal outer table is called a cephalohematoma. An
epidural hematoma can decompress through a fracture to cause a cephalohematoma or subgaleal
hematoma.
Subdural hemorrhages are due to tears of bridging veins. There is dissection of the
subdural space with dural vessels remaining close to the inner table. SDH may be due to minor
trauma or no trauma with such things such as bleeding diatheses. A SDH is usually crescentic
and, since it is not subperiosteal, crosses regions of sutures.
*
Answers: 147. True 148. False 149. True 150. True 151. False

1992
Which of the following are true regarding herpes encephalitis?
152. more commonly seen in children than adults
153. patients are most often immunocompromised
154. often hemorrhagic
155. due to Herpes zoster virus
156. involves the temporal lobes
*
Herpes encephalitis is more common in adults. In children it is secondary to HSV II
which can cause a variety of findings: calcification, microcephaly, microphthalmia, retinal
dysplasia, and encephalitis, as well as severe sequellae in survivors.
HSV I is the most common cause of herpes encephalitis in adults. It usually involves the
temporal lobes asymmetrically, the insula, the orbitofrontal region, and the cingulate gyrus. One
third is primary and 2/3 is reactivation. There is often hemorrhage (either gross or patechial).
Zoster causes a granulomatous angiitis in immunocompromised patients.
Reference: Grossman and Yousem 1994, pp. 182-183
*
Answer: 152. False 153. False 154. True 155. False 156. True
21

1993
Regarding meningiomas:
157. bright on T2
158. uncommon in the spine in children
159. located in the atria of the lateral ventricle
160. associated with overproduction of CSF
*
Meningiomas are hypo to iso on T1 to gray matter and iso to hyperintense on T2 to gray
matter. Intraventricular meningiomas typically occur around the choroid in the trigone of the
lateral venticle esp the left lat vent. p 68-73 Requisites

Answer: 157. True 158. True 159. True 160. False

1991
Regarding anencephaly:
161. most common in males
162. frequently associated with other major organ system abnormalities
163. oligohydramnios is seen early in the 2nd trimester
164. brainstem and some mesencephalon structures are present
*
Anencephaly is associated with spinal dysraphism and polyhydramnios. p 250 Req
Spinal and non CNS abnormalities and polyhydramnios is common. Anencephaly is
characterized by absence of the cranial vault, cerebral hemispheres, diencephalic structures and
their replacement by amorphous vascular-neural mass. p 883-4 Rumack
*
Answer: 161. False (F:M = 4:1) 162. true 163. False 164. True

1991
Regarding spinal meningiomas:
165. intradural extramedullary
166. more common in cervicothoracic region
167. more common in women
168. hyperintense on T2
*
Answer: 165. True 166. True 167. True 168. True
1991
Which of the following are associated with elevated alphafetoprotein levels?
169. twins
170. closed neural tube defect
171. omphalocele
*
Answer: 169. True 170. False (skin is a barrier) 171. True
22

1991
Periventricular parenchymal calcification is seen in:
172. Sturge-Weber
173. tuberous sclerosis
174. TORCH
175. neurofibromatosis
*
Sturge-Weber calcification occurs in the parenchyma. In TORCH, toxoplasmosis and
CMV do it.
*
Answer: 172. False 173. True 174. True 175. False

1991
Which of the following are midline in a child?
176. cerebellar astrocytoma
177. ependymoma
178. hemangioblastoma
179. medulloblastoma
*
The most typical appearance of a posterior fossa ependymoma on CT is that of an iso- to
hyperdense 4th ventricular mass with punctate calcifications, small cysts, and moderate contrast
enhancement. Nearly 50% will show multifocal, small calcifications; larger calcifications are
occasionally seen,
Posterior fossa astrocytoma is eccentric in location. Hemangioblastoma is hemispheric.
In younger pt with HB, t/c VHL. p 82-4 Req.

Answer: 176. false 177. True 178. False 179. True

1991
Regarding meningioma:
180. most common location is the atrium
181. spreads along CSF pathways
*
Most common location for a mening is supratentorial. In descending order, the common
locations include : parasagittal dura, convexities, sphenoid wing, CPA, olfactory groove, planum
sphenoidale. p 68 Req

Answer: 180. false 181. ???


23

1991, 1988
The following are phakomatoses:
182. neurofibromatosis
183. tuberous sclerosis
184. ataxia telangiectasia
185. Gardner’s syndrome
186. Peutz-Jeghers
*
Von Hippel-Lindau disease is an autosomal dominant disorder characterized by retinal
angiomas, cerebellar and spinal cord hemangioblastomas, angiomas of the liver and kidney,
pheochromocytomas, and cysts of the pancreas, kidney, and liver. Renal cell carcinomas also
arise more frequently in these patients.
Sturge-Weber disease is a congenital disorder characterized by angiomatosis involving
the face, the choroid of the eye, and the leptomeninges. Calcifications occur in the cerebral
cortex underlying the meningeal angiomatosis and are the most frequent CT finding. The
calcifications are gyriform in shape, and are most frequently seen in the temporoparietooccipital
regions.
Tuberous sclerosis is characterized by multiple subependymal hamartomas Giant cell
astrocytoma refers to enlarging subependymal nodules that tend to be situated near the foramina
of Monro; they are seen in 5-10% of patients with tuberous sclerosis.
Phakomatoses refer to a group of hereditary diseases of the neuroectoderm characterized
by cutaneous manifestations. Included are NF, TS, VHL, Sturge Weber. Also included in the
phakomatosis is meningiomatosis which is hamartomatous proliferation of meningeal cells via
the intraparenchymal blood vessels in the cerebral cortex AND neurocutaneous melanosis which
is characterized by cutaneous nevi and melanotic thickening. p 266-72

Answer: 182. True 183. True 184. ???false 185. False 186. False

1991
In ataxia telangiectasia, there is inability to repair chromosomal damage:
*
Answer: 187. True

1994
Which of the following commonly cause hemorrhagic metastases in the brain?
188. melanoma
189. colon
190. lymphoma
191. choriocarcinoma
192. renal cell carcinoma
*
24

Mets which have a propensity for hemmorrhage include melanoma, renal cell,
choriocarcinoma, and thyroid ca. While bronchogenic ca doesn't have a propensity for
hemorrhage, hemorrhagic lesions may be seen b/c lung ca is so common.
ref: p 423 Lee, Rao, Zimmerman- Cranial MR and CT

Answer: 188. true 189. false 190. false 191. true 192. true

1994
Which of the following exhibit dramatic clinical response to steroids?
193. cytotoxic edema secondary to infarct
194. vasogenic edema secondary to tumor
195. herpes encephalitis
196. multiple sclerosis
197. CNS lymphoma
*
Vasogenic edema results from breakdown of the blood brain varrier. Cytotoxic edema
begins as intracellular swelling secondry to any type of insult to the cell membrane usually due to
ischemia p 295 Lee, Rao
In stroke, treatment of edema is done with water restriction and agents that raise serum
osmolality such as mannitol. p 1996
With edema secondary to tumor, treatment is based on lowering intracranial pressure.
Treatment with daily doses of dexamethasone or methylprednisone reduces cerebral edema and
improves surgical prognosis. This may not control sx of obstructed ventricular system, so a
ventriculostomy may need to be placed. p 2014
HSV encephalities is Rx with IV antiviral therapy- acyclovir which has been noted to
shorten the course of infection. p 684
CNS lymphoma is Rx'ed with high dose methotrexate, glucocorticoids alone or in combo
with cyclophosphamide, doxorubicin, vincristine followed by XRT. p 2019
No effective treatment for MS is known. Therapy is directed towards reducing the acute
episode, prevent relapses or progression, and relief of Sx. In acute flare, glucocorticoid treatment
may lessen the severity, but recoverty is not improved or the extent of disability is not altered.
Can give ACTH. Immunosupressive agents like azathioprine, and cyclophosphamide have
claimed to reduce the # of relapses but there is no consensus. All tried is total lymphoid
radiation, alpha and beta interferon, diet alteration. p 2042-3
ref: Harrision's 12th ed
Answer: 193. false 194. true 195. false 196.false prob 197. true

1994
Pineal tumors include:
198. PNET
199. germinoma
200. teratoma
*Pineal region masses include pineoblastoma which is a PNET, germinoma, teratoma. Also
included are pineocytoma, chorioca, embryonal cell, glioma.
ref: p 95 , 83 Requisites
25

Answer: 198. true 199. true 200. true

1994
Regarding cavum septum pellucidum:
201. rare in newborns
202. indicates hydrocephalus
203. pathologic
*
The frontal horns and the body of the two lateral vents are separated by the septum
pellucidum. The two layers containing CSF are consistently seen in infants. In the majority, they
fuse but if they don't it is a cavum septum. It's posterior extionsion is the cavum verge. The
septum pellucidum has neural connections with the cerebral hemispheres.
p 103 Lee, Rao, Zimmerman

Answer: 201. false 202. false 203. false

1989
Regarding the appearance of Sturge-Weber syndrome:
204. intracranial calcification
205. skull thickening
206. angioma of the retina
207. intrarenal tumors
208. rhabdomyoma of the heart
*
Sturge-Weber syndrome is also known as encephalotrigeminal angiomatosis. Findings
include “port wine” nevus of the face (V1 of the trigeminal nerve), mental retardation, seizures,
leptomeningeal angiomatosis, and glaucoma, also choroidal/scleral hemangiomas. Gyriform or
tram-track pattern of calcification is the most common. It typically occurs in the parieto-occipital
region. The calcifications are deposited in gyri beneath venous angiomas and in the meninges.
The involved hemisphere is usually atrophic leading to ipsilateral calvarial thickening.
Angiomas of the retina are found in Von Hippel-Lindau.
Rhabdomyoma of the heart is found in tuberous sclerosis. By intrarenal tumors they
probably mean angiomyolipoma, which is also found in TS.
Wyburn-Mason syncdrome may be forme fruste of S-W-- have facial vascular nevus in
nerve V, retinal angioma, and midbrain AVM. p 271 Req
*
Answer: 204. True 205. True 206. False 207. False 208. False
26

1988
Which of the following are causes of intracranial calcifications?
209. Cornelia Delange
210. progeria
211. hypoparathyroidism
212. tuberous sclerosis
213. Sturge-Weber syndrome
*
Answer: 209. False 210. False 211. True 212. True 213. True

1988
Which of the following are true regarding the position of the facial nerve in the internal auditory
canal?
214. anterior to the vestibular nerve
215. above the crista falciformis
216. below the crista falciformis
217. superior to the cochlear nerve
*
The facial nerve is superior and anterior(Scott Atlas) in the IAC. So the facial nerve is
superior to crista falciformis and ant to
Bills bar.
sup
VII l VIII sup-vestibular
anterior __________ l_____________________ posterior
VIII cochlear l VIII inf vestibular
inferior

Answer: 214. True 215. True 216. False 217. true

1988
Lesions in which of the following areas can result in homonymous hemianopsia?
218. lateral geniculate
219. anterior to the chiasm
220. posterior to the chiasm
221. in the occipital lobe
*
Answer: 218. True 219. False 220. False 221. True

endhere
27

Neuroradiology Section
Select the single best answer:
1995
1. In a patient with multiple cerebral aneurysms, the most reliable finding which will accurate
localize the site of bleeding is:
a. largest aneurysm
b. irregularity of anerysm ???
c. location of the hematoma
d. vasospasm

The best indication of the aneurysm that bled is finding a hematoma with associated with
it. Ohter findings include:
ACA aneurysm- blood in the interhemispheric fissur, lat vent
MCA-- hemm in the sylvian fissure
PCA or ICA- blood in the 4th vent.
MR is useful to find hemorrhage in the wall of the aneurysm that bled. Angio findings
associated with rupture include: proximity to spasm, irregularity of aneurysm, tendency for larger
aneurysm to bleed, mass effect around aneurysm, tapering of the apex of aneurysm.
ref: p 138 Requisites

Answer: c. location of the hematoma

1993
2. A 20 y/o was weight lifting and suddenly developed confusion. He has diminished right visual
field response. The left pupil is smaller than the right. He is speaking gibberish. His brother has
been nursing a cold sore for 10 days. Which of the following is the most likely diagnosis?
a. herpes encephalitis
b. left internal carotid artery dissection
c. PICA distribution infarct
*
For an internal carotid dissection to do this, there must also be emboli since the lesions
are separated in space (3rd cranial nerve nucleus, left parietotemporal region). Sx for extracranial
dissection can present with neck and face pain, HA, horner's syndrom , cranial nerve
involvement, ischemic sx. With cervical internal carotid dissections, get Raeder's syndrome-
ptosis, miosis, HA, and preserved ipsilateral face sweating because of preserved sympathetic
fibers. Dissection can occur after blunt trauma, abrupt head turning, sports injury, direct
puncture. p 133-4 Req.
Right PICA syndrome includes right-sided ataxia, loss of pain and temperature sensation
on the right side of the face, loss of pain and temperature sensation in the left upper and lower
extremities, difficulty swallowing, loss of taste on the right, vertigo, and nystagmus.
HSV-1 causes encephalities. Sx include alteration of pt state of consciousness with
lethargy, drowsiness or stupor. Pt can have confusion, disorientation, hallucinations. Pt can have
N/V, HA. Can get aphasia, involuntary movemnets, ataxia, sensory defects, loss of retentive
memory. p 2033 Harrison's 12th ed.
*
28

Answer: b. left internal carotid artery dissection


1995
3. A 52 y/o interior decorator has just finished 2 weeks of painting a house. She presents with
loss of pain and temperature sensation on the right side of the body with loss of pain and
temperature sensation on the left side of the face. She also has ataxia and her tongue deviates to
the right. What is the most likely diagnosis?
a. Wallenberg syndrome secondary to left ??? dissection
b. Weber’s sydrome
c. central pontine infarct
d. glioma
e. Horner’s syndrome
*
Wallenberg Syndrome is lateral medullary synd. p 2080. When infarction is in the
vertebral artery distribution, the lateral medulla is most often affected +/_ the posterior inferior
cerebellum. Lateral medullary infarction can result from occlusion of vert, PICA, sup, middle,
inf lateral medullary arteries. In 70-80% of cases, get after ipsilateral vertebral artery occlusion.
Get pain, numbness, inpaired sensation over ipsilateral face, arm, trunk, leg AND contralateral
impaired pain, thermal sense over half of body. Get ataxia of limbs, falling to side of lesion,
nystagmus, horners, dysphagia, hoarsenes, paralysis of palate, vocal cords, and diminished gag.
Deviation of the tongue to the right is from 12th nerve palsy on the right which can result
from medial medullary syndrome -- occlusion of the vert or branch of the vert or lower basilar
artery.
Weber's syndrome is a result of PCA infarct Weber's is a third nerve palsy with
contralateral hemiplegia. Involves infarction of the third nerve and cerbellar peduncle. fig 351-6
p 1984
ref: Harrisons' 12th ed p 1988-9, 1984, 1987

Answer: a. Wallenberg syndrome secondary to left ??? dissection should be right vertebral
dissection b/c ipsilateral face sx and tongue deviation.

1992, 1988
4. Which one of the following vessels is affected in the Wallenberg (lateral medullary) stroke
syndrome?
a. branches off the basilar artery
b. contralateral AICA
c. ipsilateral PCA
d. contralateral PICA
e. ipsilateral PICA
* The Wallenberg stroke syndrome is loss of temperature and pain
sensation on the contralateral side and sensation of the face on the ipsilateral side. There is also
ataxia, loss of swallowing and taste sensation on the ipsilateral side, hoarseness, vertigo,
nystagmus, and ipsilateral Horner’s syndrome.
Also, see question 3 in the best answer section
Reference: Grossman and Yousem 1994, pp. 59-60
*
29

Answer: e. ipsilateral PICA

1995
5. Which one of the following infections of the brain is most commonly seen in a patient with
AIDS?
a. toxoplasmosis
b. cryptococcus
c. CMV encephalitis
d. HIV encephalitis
*
Toxo is by far the most common infection of the brain parenchyma in AIDS pts. Crypto
is at least as common but affects the meninges. In CMV see ependymitis. CMV is a common
infection in the general population.
ref: p 579 -80 Lee, Rao, Zimmerman

Answer: ??? toxo, crypto

1995
6. Which of the following is the most likely cause of an enhancing suprasellar mass in a 1-year-
old?
a. neuroblastoma metastasis
b. glioma
c. craniopharyngioma
d. pituitary adenoma
e. dysgerminoma
*
Craniopharyngioma can be seen in children and adults. They arise from metaplasia of the
squamous epithelial remnants. Calcification, nodular enhancement. Account for 6-7% of all
intracranial pediatric tumors. Angiographically, these tumors are avascular.
chiasmatic and hypothalamic gliomas can present in the suprasellar cistern. Enhancement
is variable. Seen in pt w/ and w/o NF-1. Less common than astrocytomas of cerbral hemisphere,
brainstem, or optic pathway.
Germinoma have a propensity for children and young adults. They arise from primitive
germ cells. They uniformly enhance.
Metastatic neuroblastoma frequently mets to skeleton, marrow, liver, nodes, skin. Can
met to orbit. 50% of pt with neuroblastoma are less than 2 yo. At least 15% of cancer deaths in
kids are due to neuroblastoma. In requisites, cerebral neuroblastomas are now called cotical
PNETs. Usually they are in the frontal parietal, and occipital lobes. p 97
CNS tumors account for more deaths during childhood than any other malignancy except
leukemia. Primary intracranial neoplasm accounts for 15% of neoplastic disease during
childhood. p 130
Supratentorial primary tumors include cerebral astrocytoma (37%), optic glioma (13%),
craniopharyngioma (12%), ependymoma (5%) and miscellaneous. p 133 Kirks
ref: Requisites p 320-6. Kirks, p 1010 ,144-50, 130, 133, Requisites p97
30

Answer: prob b, ?? a

1993
7. Regarding orbital pseudotumor:
a. no enhancement on CT
b. painless
c. responds rapidly to steroid and radiation therapy
d. well-defined optic nerve on CT
e. muscle insertion spared
*
Grave disease may result in thyroid ophthalmopathy, which is the most common cause of
bilateral exophthalmos in an adult. Pathologically, there is deposition of mucopolysaccharides
and infiltration by lymphocytes in the muscle bodies and retroorbital fat. Imaging reveals
bilateral enlargement of the muscle bodies, with sparing of their tendinous muscle insertions on
the globe. This is in contrast to orbital pseudotumor (myositis), wherein the tendinous insertions
are involved
Also see a True/False question in this section on the same topic
Orbital pseudotumor is a common cause of unilateral exophtalomos. Clinical features
include restriction of ocular motility, chemosis, lid swelling, pain. They usually have rapid onset
and repond to steroids. There is a chronic progressive from which responds poorly to steroids.
and is treated with chemo and XRT. In the acute case, it is a inflammatory condition that may be
an autoimmune condition involving the lacrimal gland, extraocular m, connective tissue
surrounding the dura of the optic nerve, the orbital fat, epibulbar connective tissue, and sclera.
May present as a lacrimal mass.
Tolosa-Hunt syndrome is a inflammatory condition similar to orbital pweudotumor that
involves the cavernous sinus and orbital apex. This presents with painful opthalmoplegia. p
297
ref: Requisites

*
Answer: c. responds rapidly to steroid and radiation therapy???

1992
8. The most common location of an intraventricular meningioma is:
a. atria of the lateral ventricle
b. fourth ventricle
c. posterior third ventricle
d. frontal horns
e. foramen of Monro
f. body of the lateral ventricle
*
Intraventricular meningiomas occur around chorid plexus in the trigone of the lateral
ventricle (left > right). 15% are in the 3rd ventricle and 5% are in the 4th ventricle. While
31

associated with neurofibromatosis (type II), most patients with intraventricular meningiomas
don’t have neurofibromatosis. (Grossman and Yousem 1994, p. 70)
*Answer: a. atria of lateral ventricle

1992
9. On CT myelograms, filling of the optic nerve sheath represents:
a. normal variant
b. increased intracranial pressure
c. optic nerve glioma
d. optic sheath meningioma
*
Optic nerve meningiomas typically show tram-track calcification.
*
Answer: a. normal variant (as per Dr. Goldberg)

1992
10. Which is the most likely cause of a lesion in the cerebellum with a sharp medial border at the
midline?
a. medulloblastoma
b. subacute infarction
c. hemangioblastoma
d. astrocytoma/glioma
e. metastasis
f. ependymoma
*
Only infarcts have sharp borders (representing the margin of vascular territory). All the
other lesions produce mass effect, edema, or are infiltrative.
*
Answer: b. subacute infarction

1992
11. Which of the following is the most common posterior fossa tumor in an adult?
a. hemangioblastoma
b. metastasis
c. glioma
d. medulloblastoma
e. ependymoma
*
A metastasis is the most common posterior fossa tumor in an adult (Grossman and
Yousen 1994, p. 79). However, infarct is the most common posterior fossa mass in an adult.
Lung and breast are the most common metastatic sources (followed by melanoma, RCC, and
32

thyroid). Hemangioblastoma is the most common primary tumor of the posterior fossa on an
adult.
*
Answer: b. metastasis

1992
12. What is the most likely cause of conductive hearing loss and facial nerve palsy in a trauma
patient?
a. epidural hematoma
b. vertical fracture of the petrous bone
c. transverse fracture of the petrous bone
d. longitudinal fracture of the petrous bone
*
Longitudinal fractures (parallel to the long axis) of the petrous bone are associated with
conductive hearing loss secondary to fluid in the middle ear. Cranial nerve 7 is involved in 10-
20% of injuries (this is less common than in longitudinal fractures.). Transverse fractures
(perpendicular to the long axis) are less common - complications iclude sensorineural deficits
(CN 8) and cranial nerve 7 injury (40-50%). (AFIP notes)
*
Answer: d. longitudinal fracture of the petrous bone

1994
13. A 40 year-old male is involved in a skiing accident. He is initially asymptomatic, but then
develops Horner’s syndrome, left-sided facial pain, and ptosis. Which of the following is the
most likely cause?
a. vertebral artery dissection
b. carotid artery dissection
c. internal carotid artery occlusion
d. epidural hematoma
*
Internal carotid occlusion is out. Carotid dissection is clearly the best. Jill Hunter
thought if this was not single best answer format that vertebral artery dissection might also cause
these symptoms.
Dissection of the cervicocerebral arteries cause stroke in kids and young adults. Trauma
is the most frequent cause. Horner's syndrome occurs in over half the cases. p 1981-2 Harrison's
Dissection can involve the carotid or vert. Sx include neck and face pain, HA, Horner's,
CN involvement, ischemic sx that can occur days to weeks from dissection. Ptosis, miosis, HA
with preservation of ipsilateral facial sweating (Raeder's syndrome) can occur with cervical ICA
dissection. Sympathetic fibers for sweat go with ECA and those controling the dilator pupillae
and superior palpebral muscles follow the ICA . p133-4 Requisites
*
Answer: b. carotid artery dissection
33

1994
14. A trauma patient has bilateral C6-7 facet dislocation and C6-7 cord level findings with no
sensory or motor function below that level. On the following day he regains the bulbocavernous
reflex. Which of the following is the best explanation?
a. anterior cord syndrome
b. central cord syndrome
c. lateral cord syndrome
d. posterior cord syndrome
e. total cord syndrome
f. spinal cord shock
*
The answer has to be spinal cord shock secondary to the fact that all of the other choices
represent permanent damage.
*
Answer: f. spinal cord shock

1994
15. Which of the following is the most common supratentorial neoplasm in a child?
a. PNET
b. glioma
c. ependymoma
d. hemangioblastoma
e. meningioma
*
Gliomas constitute 30% of supratentorial neoplasm in children according to Barkovich on
page 361. He further states (on page 368) that supratentorial ependymomas constitute 20-40% of
childhood ependymomas. PNETs constitute 5% of childhood supratentorial neoplasms. The
answer is most likely supratentorial gliomas.
*
Answer: b. glioma

1994
16. Which of the following is the most likely diagnosis in a demented 70 y/o man with high
intensity on T2-weighted images and focal lesions in the periventricular and subcortical white
matter?
a. Alzheimer’s
b. Pick’s disease
c. Multiinfarct dementia
d. Multiple sclerosis
*
34

Changes described above likely represent changes of small vessel ischemic disease and
therefore multi infact dementia.
*
Answer: c. multiinfarct dementia

1994, 1988
17. Which of the following is involved in otosclerosis?
a. malleolus
b. stapedius
c. cochlea
d. vestible
*
Otosclerosis (Som and Berg 1038) (aka otospongiosis) represents the deposition of
spongy vascular new bone within the otic capsule. It is another cause of sensorineural hearing
loss that is usually bilateral and seen most frequently in young to middle aged women. In the
early stages one identifies a lytic lucent erosion of the labyrinthine margins of the oval window,
the round window niche, and the cochlea. The cochlea is therefore the best choice. The middle
and baseal turns are most frequently involvoed showing areas of demineralization.
The acoustic pathway is divided into sensory (cochlear) and neural (retrocochlear)
components. Congenital or inflammatory lesions of the cochlea (cochlear otosclerosis) cause a
sensory hearing loss. Retrocochlear hearing loss has multiple etiologies; acoustic schwannoma is
the classic tumor responsible for neural hearing loss.
p 351 Req
*
Answer: c. cochlea

1994, 1992, 1988


18. Which of the following is the most likely cause of conductive hearing loss and facial nerve
palsy in a patient with acute trauma?
a. longitudinal fracture of the temporal bone
b. transverse fracture of the temporal bone
c. temporal lobe hematoma
d. epidural hematoma
*
In the patient with post traumatic conductive hearing loss (Som, p. 1032) and facial nerve
palsy typically associated with a transverse fracture of the temporal bone - transverse fractures
are only one fifth as common as longitudinal fractures and are perpendicular to the petrous ridge.
Smirniotopoulos indicates that sesorineural hearing loss due to both parts of CN VIII being
affected is a complication. Facial nerve can be involved in both but more typically in transverse
fractures (40-50%).
Longitudinal fractures usually spare the inner ear with disruption of the ossicular chain.
Longitudinal fractures are parallel to the petrous ridge. The facial nerve is affected in 10-20%,
but the eighth cranial nerve typically remains intact, resulting in a conductive hearing loss.
These represent the classical teaching and like all other things is far from the reality with
oblique fractures being the most common.
35

(Also, see question 34 regarding fractures of the petrous bone.)


References: AFIP notes (Smirniotopoulos); Seminars in US, CT, and MR 10:262-279, 1989;
Latchaw, 2nd edition, 1991, pp. 178-179, 203-265; Harnsberger, pp. 325-326; Barkovich, pp. 68-
71
*
Answer: a. longitudinal fracture of temporal bone

1995
19. Which of the following structures is the most medial structure in the cavernous sinus?
a. trigeminal nerve
b. cranial nerve 6
c. optic nerve
d. fourth cranial nerve
*
In the cavernous sinus lie the CNIII- which is in the superolat portion, CNIV- which lies
just below III, V-1, V-2, VI- which lies just lateral to the carotid. p 51 Requisites
The optic nerve is not in the cavernous sinus.
The most medial according to the fig 641 in Clemente is III, the oculomotor nerve, then
abducens-VI.
ref: Requisites p 51, Clemente fig 641

Answer: b

1993
20. The most common spinal cord tumor in a child is:
a. astrocytoma
b. hemangioblastoma
c. ependymoma
d. hemangioma
*
Astrocytoma comprises 60% of intramedullary tumors in a child.
Hemangioblastomas account for only 3% of intramedullary spinal cord tumors, and 30%
of these are seen with von Hippel-Lindau syndrome. Purely intramedullary lesions are associated
with cyst/syrinx formation in two-thirds of cases - the presence of a holocord cyst, with only
small areas of enhancement, should raise the possibility of hemangioblastoma.
Ependymoma comprises 30% of intramedullary tumors in a child. They are the most
common primary tumors to arise in the lower spinal cord, conus medularis, and filum terminale.
One subtype of ependymoma, the myxopapillary form, is particularly common in the conus and
filum; it is a mucin-producing tumor, which is prone to bleeding - sometimes presenting as
unexplained subarachnoid hemorrhage.
Hemangioma is the most common benign lesion of the vertebral body in a child. Squid
fact: Cobb’s syndrome (rare) is cutaneous malformations, vertebral hemangiomas, and
hemangiomas of the spinal cord.
36

In kirks p 231 table 3-10, a listing of primary neoplasms of the cord is listed with
astocytoma being the most common.
*
Answer: a. astrocytoma

1993
21. The most common primary cerebellar tumor in a child is:
a. hemangioblastoma
b. medulloblastoma
c. cerebellar astrocytoma
d. ependymoma
*
Hemangioblastoma is the most common infratentorial tumor in an adult.
Medulloblastomas (one of the PNET tumors) are one of the most common posterior fossa
masses in the pediatric population, accounting for more than one third of posterior fossa
neoplasms and 50% of cerebellar tumors in children (Grossman & Yousem, 1994, p. 85) These
tumors are usually seen in the midline arising from the superior velum of the fourth ventricle.
Patients usually have hydrocephalus. On nonenhanced CT the lesions are hyperdense well-
circumscribed masses. Subarachnoid seeding may be present.
Borkovich claims that there are slightly more cerebellar astrocytomas in most series. ???
Quoting Lee, Rao, & Zimmerman 1992, p. 389, “Depending on the series,
medulloblastoma or cerebellar astrocytoma is the most common primary neoplasm of the
pediatric posterior fossa.”
According to Grossman and Yousem (1994), pilocytic astrocytomas are the most
common infratentorial neoplasm in children - 20% are in the brainstem and 40% are cerebellar.
Ependymoma is usually associated with the 4th ventricle. They have a greater incidence
of calcification (40-50%) than other posterior fossa pediatric neoplasms.
*
Answer : probably b. medulloblastoma

1994, 1993, 1988


22. A patient has palsies involving cranial nerves III, IV, V1, V2, and VI. What is the location of
the lesion?
a. cavernous sinus
b. superior orbital fissure
c. optic canal
d. inferior orbital fissure
e. brainstem
* Lesions in the cavernous sinus can affect: III, IV, V1, V2, and VI
Lesions in the superior orbital fissure can affect: III, IV, V1, VI, the orbital branch of the
middle meningeal artery, the sympathetic nerve, the recurrent meningial artery, and the superior
ophthalmic vein.
Lesions in the optic canal can affect the optic nerve and the ophthalmic artery.
Lesions in the inferior orbital fissure can affect the infraorbital artery, vein, and nerve
(branch of V2)
37

Lesions in the brainstem can affect: cranial nerves I-IV if they arise from the midbrain or
above, V-VIII if they arise from the pons, and IX-XII if they arise from the medulla.
On the question in 1994, 52 and 53 were normal and since 52 and 53 are normal, the
anterior cavernous sinus is the most likely culprit. (superior orbital fissure???)
*Answer: a. cavernous sinus

1993, 1990, 1989


23. Which of the following is not associated with tethered cord and spinal dysraphism?
a. diastematomyelia
b. myolipoma
c. hydrosyringomyelia
d. thickening of the filum terminale
e. myelolipoma
*
Spinal dysraphism basically means a group of spinal anomalies with the common feature
of imperfect union of midline structures. Tethered cord is a spinal cord that is held in an
abnormally low position by a fibrous band, bone spur, or mass.
Diastematomyelia is a longitudinal split in the spinal cord, conus medullaris, or filum
terminale associated with bony abnormalities including spina bifida. It also is associated with
tethering of the conus medullaris (below L2 in 76%).
Hydrosyringomyelia is a cavity in the spinal cord. Most are associated with congenital
spinal and hindbrain abnormalities.
Thickening of the filum terminale (= tethered spinal cord syndrome) is associated with
midline bony defects.
Myelolipoma is lipoma(s) of the filum terminale. This is another form of spinal
dysraphism and tethering of the spinal cord. These lesions have fat on CT or MR.
Spinal lipomas are masses of fat and connective tissue that are at least partially
encapsulated and have definite connection to the spinal cord and meninges. They are considered
in three major groups: intradural lipomas, lipomyelomeninigoceles, and lipomas of the filum
terminale. In patients with lipomyelocele there is no substantial meningocele component and the
lipoma usually extends into the canal through an occult spina bifida. The lipoma is attached to
the dorsal surface of the neural tissue and is covered by intact skin. The spinal cord may assume
different shapes and associated tethering may be present.
Fibrolipomas frequently occur in the filum terminale, and may be considered a normal
variant if not associated with tethering of the spinal cord or neurologic dysfunction. These
developmental lesions arise secondary to the persistence of caudally located pleuripotential cells
that differentiate into fat. Fibrolipomas of the filum may be intradural or extradural, or both;
extradural lesions tend to be diffuse.
*
Answer: b. myolipoma

1993
38

24. A 35 y/o male sustained blunt trauma to the neck two weeks prior to development of
amaurosis fugax and Horner’s syndrome. Which of the following is the most likely diagnosis?
a. aneurysm of the internal carotid artery
b. pseudoaneurysm of the internal carotid artery
c. aneurysm of the vertebral artery
d. pseudoaneurysm of the vertebral artery
*
aneurysm = focal dilatation of an artery
pseudoaneurysm = organized hematoma from a vessel that has bled.
*
Answer: b. pseudoaneurysm of the internal carotid artery

1994, 1993
25. A herniated disc at L4-5 most commonly impinges on which spinal root?
a. L4
b. L5
c. S1
*
The spinal cord contains 8 cervical, 12 thoracic, 5 lumbar, 5 sacral, and 1 coccygeal
paired spinal nerves. C1 exits above the C1-2 interspace, so C2 exits between C1 and C2, etc.
The vertebral bodies in the lumbar spine are tall and the nerve roots leave the thecal sac right
under the pedicle above the interspace. Therefore, herniations in the lumbar region strike the
root exiting below the interspace. However, a far lateral disc herniation could impinge on the
root exiting above the interspace (L4 in this case - also the answer to the question on the 1994
boards which asked about a far lateral herniation) and a very large herniation could impinge on
both the root above and the root below.
ref: p 448 Requisites
*
Answer: b. L5 (but see above for answer to 1994 Boards question)

1993
26. Which of the following modalities is most useful to distinguish recurrent glioblastoma from
radiation-induced necrosis?
a. mass effect on CT
b. gadolinium-enhanced MR
c. 18F-deoxyglucose PET scan
d. contrast-enhanced CT
*
Radiation necrosis is usually a late effect of treatment. The late injury may be focal or
diffuse. XRT necrosis favors the tumor site. CT or MR may demonstrate a mass lesion
associated with edema (low attenuation on CT or high signal on T2) which usually enhances.
39

Diagnosis of XRT necrosis is made by surgical biopsy but may be suggested by PET.
Residual or recurrent tumor has increased activity on PET (XRT necrosis has low activity). The
overall accuracy of PET is 85%.
*
Answer: c. 18F-deoxyglucose PET scan

1993
27. A diabetic patient has a CT which shows enhancement in the inferior frontal lobes. Which of
the following is the most likely diagnosis?
a. infarct
b. PML
c. lymphoma
d. fungal encephalitis
e. herpes
*
Progressive multifocal leukoencephalopathy (PML) is a demyelinating disease with viral
etiology (papovavirus). It is associated with immunosuppression. It can occur anywhere in the
brain (usually in the subcortical white matter, but can involve the cortex - in contrast to HIV
encephalitis which is characteristically periventricular). It can be solitary or multifocal. It is low
intensity on T1 and high intensity on T2. Usually there is no enhancement.
Lymphoma is not uncommonly seen in immunosuppressed patients (AIDS, post-
transplant, etc.). There is a marked predilection for the basal ganglia, cerebellar hemispheres,
thalamus, brain stem, corpus callosum, and subependymal region. Primary CNS lymphoma
occuring in AIDS patients is a highly aggressive malignancy which is frequently necrotic and
surrounded by vasogenic edema. The central necrotic portion is hypodense on CT and the
periphery enhances with contrast. In contrast, lymphoma occuring in patients who do not have
AIDS is more typically a solid, hypercellular tumor and is therefore often hyperdense on
noncontrast CT scans. Following gadolinium or iodinated contrast, enhancement is typically
uniform. Primary CNS lymphoma virtually never calcifies.
[fungal encephalitis] Mucormycosis affects patients with abnormalities of host defenses,
particularly patients with diabetes and ketoacidosis. The fungus is usually inhaled and rapidly
destroys nasal mucosa. It may then spread into paranasal sinuses (with or without bone
destruction), orbit, skull base, or may extend throught the cribriform plate and involve the
anterior cranial fossa. There is a high frequency (50% in some series) of intracranial extension.
With intracranial extension, there are low density abnormalities in the anterior cranial fossa
(these may in fact be present in any part of the brain). One can also see mass effect and
enhancement.
Herpes virus Type I is responsible for encephalitis in adults. One-third of cases are due to
primary infection (usually <18 y/o) and 2/3 are due to reactivation. Herpes attacks the temporal
lobes, insula, orbitofrontal region, and cingulate gyrus. MR examination done during the first
five days of infection shows high signal on PD and T2 in the temporal and inferior frontal lobes
with increasing mass effect. Linear, subarachnoid, and gyriform enhancement may be seen.
*
40

Answer: d. fungal encephalitis

1993, 1988
28. Which of the following is most highly associated with peripheral aneurysms of the MCA?
a. mycotic aneurysm
b. polyarteritis nodosa
c. Berry aneurysm
d. Takayasu’s arteritis
*
A mycotic aneurysm is often secondary to endocarditis with septic emboli to the vasa
vasorum and secondary destruction of the vessel wall - all that is left is the intima. The
aneurysms may be saccular or fusiform. The aneurysms tend to be peripheral in the MCA
distribution.
Polyarteritis nodosa is a multisystem disease with necrotizing inflammation of muscular
arteries. There is CNS involvement in 45% (usually late). There are no specific findings.
Berry anuerysms, or saccular aneurysms, are congenital weaknesses in the media and
elastica of the arterial wall. The occur at branching point where the parent vessel is curving.
Takayasu’s arteritis tends to involve large vessels coming off the arch. Aneurysms are
not a typical feature. Generalized vasculitis may lead to infarcts.
*
Answer: a. mycotic aneurysm

1993
29. Which of the following is true regarding intraventricular meningioma?
a. it most commonly involves the atrium of the lateral ventricle
b. it frequently seeds the CSF
c. in results in increased CSF production
d. it obtains its blood supply from the external carotid artery
*
Intraventricular meningiomas typically occur around the choroid plexus in the trigone of
the lateral ventricle and have a distinct propensity for the left lateral ventricle. They calcify in
45-68% of cases. Blood supply to an intraventricular meningioma is usually via choroidal
vessels.
*
Answer: a. it most commonly involves the atrium of the lateral ventricle

1994, 1992, 1988


30. Which of the following is least likely to be a posterior fossa tumor in a child?
a. brainstem glioma
b. choroid plexus papilloma
c. ependymoma
d. astrocytoma
e. PNET
41

f. hemangioblastoma ???
*
The most common posterior fossa lesions in kids are pilocytic astrocytoma and PNET
(medulloblastoma) which each account for 1/4-1/3. Brainstem glioma and ependymomas are the
next most common (each about 1/6). While choroid plexus papillomas occur most frequently in
kids (35% of cases occur in kids less than 5 y/o) and are in the 4th ventricle, they account for
only 3% of childhood tumors. (Grossman and Yousem 1994, p. 78)
From Jill and Josh:
Medulloblastoma (30-40% of posterior fossa neoplasms in children (Barkovtch, p. 324))
and cerebellar astrocytoma are the most common. Ependymoma represents 15% of posterior
fossa neoplasms (Barkovich, p.334). Osborne states that choroid plexus papilloma almost
always arises in the lateral ventricles in children (In adults, the 4th ventricle is the most common
location.). These are uncommon tumors with a median age of 26 months. Symptoms are
generally due to hydrocephalus. The answer most likely is choroid plexus papilloma.
*
Answer: b. choroid plexus papilloma

1993
31. Which of the following is the most common cause of a peripheral MCA microaneurysm?
a. polyarteritis nodosa
b. septic emboli
c. intravenous amphetamine
d. berry aneurysm
e. atherosclerosis
*
Mycotic aneurysms are peripheral to the 1st bifurcation of a major vessel in 64% and are
often located near the surface of the brain, especially over the convexities. (e.g., often just distal
to MCA trifurcation in the inferolateral Sylvian fissure.)
Polyarteritis nodosa may involve all organs, but mainly the kidney (85%), heart (65%),
and liver (50%).
Intravenous amphetamine can yield a CNS vasculitis and subsequent microaneurysms,
but this is rare.
*
Answer: b. septic emboli

1992
32. A patient with spinal cord trauma has headache, diaphoresis, and blood pressure fluctuations
after intravesical instillation of 150 cc of contrast. All symptoms are relieved by insertion of a
Foley catheter. What is the level of injury?
a. cervical spine
b. T8-9
c. T12-L4
d. S1-2
42

e. S3-4
f. cauda equina
*
Symptoms of autonomic dysreflexia occur with spinal cord lesions above T5 (level of
greater splancnic nerve) secondary to alpha-adrenergic response below the spinal cord lesion
uncontrolled by central centers. Therefore, there is a rapid rise in blood pressure, pallor below
the level of the lesion, flushing and diaphoresis above the level of the lesion, headache, coma,
and potentially death. Activation of the carotid sinus baroreceptors can cause bradycardia.
Reference: Dahnert, p. 363.
*
Answer: a. cervical spine

1992
33. Which of the following is the most likely diagnosis in a head trauma patient who loses
consciousness, recovers to have a lucid interval, and again loses consciousness?
a. subdural hematoma
b. epidural hematoma
c. contusion
d. subarachnoid hemorrhage
*
A lucid interval is seen in 40% of patients with epidural hematoma.
General guidelines following head trauma:
1) conscious ---> unconscious = nonspecific
2) unconscious + doesn’t wake up = DAI
3) unconscious --> lucid --> unconscious = epidural
DAI injury is due to shearing and is most commonly seen in the gray-white junction, the
corpus callosum, and basal ganglia.
*
Answer: b. epidural hematoma

1989, 1990
34. There is a single ventricle and midline fusion of the thalami. What is the most likely
diagnosis?
a. holoprosencephaly
b. encephalocele
c. infarct
d. hydrocephalus
*
Holoprosencephaly represents a failure of normal development of the forebrain.
Complex facial and craniocerebral abnormalities are associated. There are three classifications:
alobar, semilobar, and lobar.
43

In the alobar form, the cerebral hemispheres are absent. A crescent-shaped monoventricle
is present with a thin rim of cortical tissue. The thalami are fused. The septum pellucidum and
interhemispheric fissure are absent.
In all three types, the septum pellucidum and olfactory bulb are absent. In the alobar and
semilobar forms, midline facial anomalies are common.
Meningoencephalocele is a disorder of midline closure commonly seen in the occipital
region. It represents herniation of the meninges-containing CSF through a congenital defect in
the skull vault. In an encephalocele, contents include brain tissue. The most common site is the
occipital region.
*
Answer: a. holoprosencephaly

1993
35. A patient with unilateral field cut and medial rectus paralysis has a lesion in:
a. cavernous sinus
b. cerebellopontine angle
c. occipital lobe
d. medulla oblongata and the pons
e optic chiasm
*
The lesion has to be before the optic chiasm. CNIII supplies extraocular muscles except
lateral rectus and superior oblique. The nucleus is in the midbrain just posterior to the red
nucleus and anterior and superior to the aqueduct. It goes thru the interpeduncular cistern to
between the PCA and superior cerebellar arteries lateral to the PCOM. Thru the caverous sinus
thru the superior orbital fissure. The optic nerve goes back to the lateral geniculate in the
pulvinar of the thalamus and back thru the geniculocalcarine tract.
ref; Req p 45, see figure 2-14 p 44

Answer: d. medulla oblongata and the pons

1991
36. In a patient with complete proximal ICA occlusion, an ipsilateral injection is performed and
filling of the distal ipsilateral ICA is seen. What is the supply of blood to the distal ICA?
a. anterior communicating artery
b. retrograde flow via the ophthalmic artery
c. from the posterior circulation
*
The intracranial extracavernous internal carotid artery gives rise to several important
branches:
1. superior hypophyseal artery
2. ophthalmic artery (anterior falx artery and, occasionally, the middle meningeal artery arise
from it)
3. posterior communicating artery (one or both are hypoplastic in 20% of cases)
44

4. anterior choroidal artery (divides into proximal (cisternal) and distal (plexal) segments) - it is
an important supply to the choroid plexus of the temporal horn of the lateral ventricle.
5. small unnamed branches to the hypothalamus, optic tract, and optic chiasm
The ophthalmic artery is an important source of collateral flow in cases of cervical ICA
occlusion.
*
Answer: b. retrograde flow via the ophthalmic artery

1991
37. Which of the following is the most convincing evidence against multiple sclerosis in a
patient?
a. one neurologic event, normal physical exam
b. normal MR of the brain
c. internuclear ophthalmoplegia
*
Multiple sclerosis is a demyelinating disease of unknown etiology which most frequently
affects young adults. The classic lesions seen on MR are plaques, which occur commonly in the
periventricular white matter, internal capsule, corpus callosum, pons, and brachium pontis. They
are best seen on long TR sequences.
*
Answer: ???

1991
38. A 30 y/o female had a severe headache 6 days ago. She has a known Berry aneurysm and
now presents with an acute headache and one episode of nausea without vomiting. What is the
most likely diagnosis?
a. recurrent subarachnoid hemorrhage
b. communicating hydrocephalus
*
Intracranial aneurysms are the most common atraumatic cause of subarachnoid
hemorrhage. Only about 10% occur in the posterior fossa, the majority of these arising in the
basilar tip. At least 20% are multiple. By definition, aneurysms larger than 2.5 cm are termed
giant. Although the vast majority of berry aneurysms occur in isolation, certain diseases
predispose to their formation. These include autosomal dominant polycystic kidney disease
(???6% incidence of aneurysm), AVM, fibromuscular dysplasia, coarctation of the aorta, and
collagen vascular disease.
*
Answer: a. recurrent subarachnoid hemorrhage

1991
39. What is the most likely diagnosis if a 50-60 HU density is seen in the brain and it has
peripheral enhancement?
a. aging hematoma
45

b. high grade astrocytoma


*
Answer: a. aging hematoma

1988
40. A 45 y/o woman has unilateral papilledema and calcification of the junction of the optic
nerve and posterior globe. What is the most likely diagnosis?
a. retinoblastoma
b. choroidal osteoma
c. Drusen
d. melanoma
*

Lesions at termination of the optic nerve:


1) Calcified
a. retinoblastoma
b. astrocytic hamartoma (tuberous sclerosis)
c. choroidal osteoma
d. Drusen
e. scleral calcifications
2) Noncalcified
a. melanoma
b. metastases
c. choroidal hemangioma

Retinoblastoma typically presents as leukokoria or strabismus. It is the most common


tumor of the globe in children. Bilateral involvement (30% of cases) is associated with the
heritable form of retinoblastoma, is usually multifocal, and tends to be diagnosed in younger
children (average = 8 months, 98% in kids less than 3 y/o) than unilateral involvement (average =
25 months). The presence of bilateral retinoblastoma should prompt the search for a pineal cell
neoplasm (trilateral retinoblastoma). Unilateral disease is often unifocal and sporadic.
Metastases are extremely infrequent. Enucleation is curative (ocular preservation is attempted if
less than half of the retinal surface is affected and no adjacent structures are affected.)
Melanoma are usually hyperintense on T1 and hypointense on T2. IT is associated with
retinal detachment, subretinal proteinaceous effusions.
Choroidal osteoma is benign tumor in young women. The lesion are usually unifocal,
demonstrate ossification, and are juxtapapillary. Punctate calcification in the posterior globe on
the temporal side of the nerve is usually seen. The ddx includes amelanotic melanoma, choroidal
hemangioma, mets, scars, resolving hemorrhage. p 287
Drusen is made of a mucoprotein matrix. It occurs in 2% of the population. Can be
familial and typically bilateral. It is located at the junction of the optic nerve and globe. CT
demonstrates punctate calcifications. Normal course may be progressive enlargement. p 290
Ref: Requisites, p 287, 290
*
46

Answer: choroidal osteoma prob

endhere

Neuroradiology Section
The following are matching questions:

1995
The following had a relatively primitive drawing where we had to label the following veins:
1. basal vein of Rosenthal
2. inferior sagittal sinus (sss on diagram but not labeled)
3. vein of Galen
4. inferior cerebral vein

Answer: ???
xxxxxxxxxxxxxxxxxxx

1995
5. mamillary bodies
6. hippocampus
7. caudate nucleus

a. mesial temporal sclerosis


b. Wernicke’s syndrome
c. Huntingdon’s chorea
d. Parkinson’s disease
e. Dilantin therapy
*
Huntington's is characterized by chorieform movements with rapid onset dementia. It's
autosomal dominant, predominantly involving males. The hallmark is atrophy of caudate
nucleus and subsequently the putamen. The later stages involves temporal and frontal lobes.
Parkinson's is a movement disorder associated with cogwheel type rigidity. The disease
involves the substantia nigra. Inlater stages, it involves the sorpus striatum and globus pallidus. p
270
p 273-4 Lee, Rao, Zimmerman.
47

Dilantin causes atrophy of the cerebellum.

Answer: 5. b 6. a 7. c

1995
8. associated with orbital cellulitis
9. most commonly associated with mucocele
10. most common location of mucus retention cyst
11. most common location of polyps

a. maxillary sinus
b. frontal sinus
c. ethmoid sinus
*
Orbital cellulitis in kids and dolescents is most often assocated with paranasal sinuses.
Increased pressure in the sinus is transmitted to the orbit and causes preseptal edema. Septic
thrombophlebitis leads to cellulitis and may lead to orbital cellulitis.--- ethmoid and frontal.
p128-9 Acute ethmoiditis can result in infection traversing the thin plate of bone causing orbital
cellulitis. p 1098 Harrison's
Mucocele are most common in the frontal and ethmoid sinuses with sphenoid sinus the
least common. The most frequent location for a mucocele is the frontal sinus (60%). p 182 Lee
Rao.
Retention cyst are a result of obstruction of small sermucinous glands usually in the
maxillary sinus..
One manifestation of allergic sinusitis is sinonasal polyposis. The lesion although benign
may demonstrate fairly aggressive bony distortion.
ref: Requisites, p 369, 128-9, Harrison's p 1098, Lee, Rao- p 182

Answer: 8. c 9. b 10. a 11. ????

1995
12. brachycephaly
13. scaphocephaly
14. macrocrania
15. microcrania

a. Apert’s disease
b. Trisomy 13
c. Chiari II malformation
d. sagital suture synostosis
*
Macrocrania is associated with craniosynostosis. Microcrania is associated with trisomy
13. Universal craniosynostosis leads to microcrania. Table 2-7 in Kirks p 75
48

Premature sagittal suture closure produces a long narrow skull termed dolichocephaly or
scaphocephaly. p 84
Brachycephaly is when the skull is short in AP diameter and can be seen in coronal
craniosynostosis.
Apert syndrome leads to secondary craniosynostosis and hypertelorism. p 83
Hypertelorism is greater interorbital distance. Bilateral coronal craniosynostosis can cause
hypertelorism. p68
Arnold -Chiari malformation leads to increased intracranial pressure. In neonates and
infants, increased pressure leads to sutural diastasis. In pt with closed sutures, get increased
inner table markings and sellar erosion. p 77

Answer: 12. ?a 13. d 14. prob c ,could also be a,d 15. b

1995, 1989
Matching:
16. V1 (89)
17. V2 (95, 89)
18. V3 (95, 89)
19. fascial nerve (95)
20. spinal accessory nerve (95)

a. superior orbital fissure


b. inferior orbital fissure
c. foramen ovale
d. foramen rotundum
e. formanen spinosum
f. jugular foramen
g. stylomastoid foramen
*
Answer: 16. a 17. b 18. c 19. g 20. f

1995, 1992
21. elderly with cortical and subcortical hemorrhage
22. increased risk of bleeding over next 2 weeks
23. progressive bleeding over 24-48 hours
24. predilection for frontal and anterior temporal lobes
25. putamenal infarct or bleed

a. amyloid angiopathy
b. trauma
c. venous angioma
d. mycotic aneurysm
e. hypertension
49

*
2/3 of mycotic aneurysms rupture within the first 3 weeks of diagnosis of endocarditis.
For aneurysms in general, 20-50% rebleed within the first 2 weeks of subarachnoid hemorrhage.
Most contusions in trauma occur in the frontal and anterior temporal lobes secondary to
the rough surface of the anterior and middle cranial fossa and sharp border of the planum
sphenoidale.
Hypertensive bleeds tend to occur in the basal ganglia.
Venous angioma:
- the most common vascular abnormality at autopsy
- usually asymptomatic
- cerebellar and frontal
- usually solitary
*
Answer: 21. a 22. d 23. b 24. b 25. e

1994
26. Facial nerve
27. Trigeminal (53)
28. vestibular

a. stapedius
b. tensor tympani
c. buccinator
d. posterior digastric
e. anterior digastric
*
Buccinator and stapedius are innervated by facial nerve.
A branch of the mandibular nerve (53) supplies the tensor tympani
The mylohyoid and anterior belly of the diagastric combine with the extrinsic tongue
muscles to form the floor of the mouth and are supplied by CN 12.
The posterior belly of the digastric is supplied by the facial nerve and the anterior belly is
supplied by a branch of the mandibular division of the trigeminal-v-3. p 720 Snell
*
Answer: 26. a and c and d 27. b and e 28. ???

1994
Regarding the Phacomatoses:
29. giant cell astrocytoma
30. optic nerve glioma
31. bilateral acoustic neuromas
32. hemangioblastoma
33. Shagreen patches
50

a. Neurofibromatosis, Type I
b. Neurofibromatosis, Type II
c. von Hippel Lindau disease
d. Tuberous sclerosis
*
Neurofibromatosis occurs in approximately 1 in every 3300 live births.
Neurofibromatosis - I (von Recklinghausen’s disease)
- autosomal dominant (abnormality is on chromosome 17)
- diagnostic criteria include six or more cafe-au-lait spots, axillary or inguinal freckling, two (or
more) neurofibromas, optic glioma (most common CNS neoplasm - occurs in 30-90% and is
bilateral in 10-20%), iris hamartoma, distinctive osseous lesions, or a first-degree relative with
NF-1
- hypertension may be found with renal artery fibromuscular dysplasia
- neurofibromas can occur in the urinary and gastrointestinal tracts
- increased incidence of all types of neoplasms
- malignant transformation of the neurofibroma into neurofibrosarcoma occurs in about 5%
- lateral thoracic meningoceles are a common manifestation of spinal neurofibromatosis; most
authors believe that they are secondary to menigeal dysplasia (dural ectasia). 80% of patients
with lateral thoracic meningoceles have neurofibromatosis.
- a plexiform type of neurofibroma is virtually pathognomonic of neurofibromatosis

Neurofinbromatosis - II
- autosomal dominant (abnormality is on chromosome 22)
- bilateral 8th cranial nerve tumors or
- unilateral 8th cranial nerve tumor or neurofibroma, menigioma, glioma, Schwannoma, or
capsular cataract and a first degree relative with NF-2

29. giant cell astrocytoma


30. optic nerve glioma
31. bilateral acoustic neuromas
32. hemangioblastoma
33. Shagreen patches

a. Neurofibromatosis, Type I
b. Neurofibromatosis, Type II
c. von Hippel Lindau disease
d. Tuberous sclerosis

Answer: 29. d 30. a 31. b. 32. c. 33. d

1993
34. Commonly occurs in the horizontal segment of MCA
35. Typically spares the bifurcation of vessels
36. in the ICA occurs near the skull base
51

37. vertebral artery at approximately C1-C2


38. vertebral artery at the base of the skull

a. embolus/thrombus
b. vasospasm
c. dissection
d. mycotic aneurysm
e. ulcerated plaque
*
Dissection usually affects the extracranial great vessels but can occasionally occur in
intracranial vessels. Symptoms differ in that extracranial dissection can present with neck and
face pain, headache, Horner’s syndrome, cranial nerve involvement, and ischemic symptoms that
occur days to weeks after the dissection. The association of ptosis and miosis with headache and
preservation of ipsilateral facial sweating (Raeder’s syndrome) can occur with cervical internal
carotid dissection, because the sympathetic nerve fiber for sweat follow the external carotid
artery whereas those controlling the dilator pupillae and superior palpebral muscle follow the
internal carotid artery. (Grossman & Yousem 1994, pp. 133-134)
*
Answers: 34. d 35. b 36. c 37. c (in vert.) 38. ???

1994, 1992, 1989


39. Internal auditory canal
40. trigone of the lateral ventricle
41. inferior colliculus

a. anterior/inferior cerebellar artery


b. posterior choroidal artery, lateral branch
c. anterior choroidal artery
d. precentral cerebellar vein
e. PICA
f. superior cerebellar artery
*
AICA is IAC. The internal auditory artery can arise from the basilar artery or more
frequently from the AICA. It enters the internal auditory canal with cranial nerves 7 and 8.
The anterior choroidal artery freely anastomosis with the posterior choroidal artery - as a
result it supplies the choroid of the inferior temporal horn and atrium. The posterior choroidal
artery supplies the choroid of the posterior temporal horn, atrium, and body of lateral ventricle.
Posterior and anterior choroidal branches supply choroid (Osborne, pp. 125-26).
The inferior colliculus typically is supplied by the PCA, less commonly by the posterior
lateral choroidal artery. It drains via the precentral vein. (“The superior vermian branch of the
superior cerebellar artery supplies tiny branches to the inferior colliculus”?)
*
Answer: 39. a 40. b 41. d, f???
52

1992
42. conductive hearing loss
43. incontinence
44. hoarseness

a. ossicular dislocation
b. meningioma at the foramen magnum
c. glomus jugulare tumor
d. glomus tympanicum tumor
*

Jugular tumors may invade adjacent nerves in the jugular foramen and around the jugular
- thus involving cranial nerve 10 and causing hoarseness.
The glomus tympanicum tumor is a paraganglioma arising at the cochlear promontory of
the middle ear cavity. It presents with pulsatile tinnitus and/or a vascular tympanic membrane.
Since it is a highly vascular tumor, intense enhancement occurs after CT or MR contrast
administration (or signal voids on MR before contrast administration). The tympanic branch of
the ascending pharyngeal artery is the major source of blood supply to the glomus tympanicum
and is therefore often enlarged when a tumor arises there.
*
Answer: 42. a 43. b 44. c

1994, 1989
45. ring enhancing lesion that is hyperdense centrally on CT
46. ring enhancing lesion with a thick wall and an inner surface that is shaggy

a. lymphoma
b. highly aggressive glioma
c. hematoma at two weeks
d. infarct at two days
*
Hyperdense lesion centrally (this is actually quite tough) most likely represents a two
week old hematoma. Contrast enhancement around the periphery of the hematoma usually
develops at 7-9 days after the bleed.
Thick wall is excellent for glioma and should lead you away from things like abscess
(classical teaching is 5 mm or thicker is glioma).
Infarcts begin to enhance after the first week after the insult. This is due to improved
collateral circulation.
Nice mnemonic for contrast-enhancing ring lesions = MAGIC DR.
M = metastases, MS
A = abscess/cerebritis
G = glioma/granuloma
53

I = infarct
C = contusion
D = demyelination (active)
R = resolving hematoma or due to radiation
Reference: AFIP notes (Smirniotopoulos)
*
Answer: 45. c 46. b

1991
47. Organ of Zuckerkandel
48. Arnold Chiari II
49. Dandy Walker cyst

a. most common location of extraadrenal pheochromocytoma.


b. hypoplasia or aplasia of corpus callosum
c. enlarged posterior fossa
*
The Dandy-Walker malformation consists of cystic dilatation of the 4th ventricle,
agenesis of the cerebellar vermis, and marked enlargement of the posterior fossa. Hydrocephalus
develops in 75% of patients; dysgenesis of the corpus callosum occurs in 20-25% of patients.
*
Answer: 47. a 48. b 49. c

1991
50. large acoustic neuroma
51. hemangioblastoma

a. hearing loss
b. brain and spinal cord
*
Answer: 50. a 51. b

1994, 1990, 1989


52. meningomyelocele
53. normal 4th ventricle
54. vermian hypoplasia
55. tonsilar herniation
56. absence of corpus callosum

a. Chiari I
b. Chiari II
c. Chiari III
54

d. Chiari IV
*
The Arnold Chiari malformations are divided into four types:
Type I - tonsilar invagination through the foramen magnum into the spinal canal with
variable degrees of cerebellar displacement (tonsilar ectopia). There is normal position of the 4th
ventricle. Syringohydromyelia is an associated finding. Two components are necessary for the
diagnosis - tonsilar herniation and syringohydromyelia.
Type II - dysgenesis of the hindbrain with caudally displaced 4th ventricle and caudal
elongation of the medulla and vermis. Concurrent anomalies include: myelomeningocele,
mesencephalic beaking (tectal beaking), enlarged massa intermedia, accessory anterior
commissure, absence of the corpus callosum (which manifests as elevated third ventricle), and
partial absence of the falx. Abnormalities of the bony vault include Luckenshadel (or lacunar)
skull (appears as “soap bubble” lucencies in the upper calvaria - it is a dysplasia of bone and
underlying dura and can be seen with any cause of meningocele and myelomeningocele) which
regresses after about 6 months of age, scalloping of the petrous bone and clivus (older children),
and enlargement of the foramen magnum (older children).
Type II - is a rare lesion that consists of herniation of cerebellum into a high cervical or
occipital encephalocele.
Type IV - consists of extreme cerebellar hypoplasia without associated displacement.

52. meningomyelocele
53. normal 4th ventricle
54. vermian hypoplasia
55. tonsilar herniation
56. absence of corpus callosum

a. Chiari I
b. Chiari II
c. Chiari III
d. Chiari IV

*
Answer: 52. b 53. a 54. d 55. a 56. b

1988
57. craniopharyngioma
58. pinealcytoma
59. cerebellar astrocytoma

a. anterior 3rd ventricle


b. tuber cinereum
c. posterior 3rd ventricle
d. hydrocephalus
*
55

Cerebellar pilocytic astrocytoma often infiltrate the 3rd ventricle and present with
hydrocephalus.

Answer: 57. a 58. c 59. d

You might also like